Last visit was: 25 Apr 2024, 01:00 It is currently 25 Apr 2024, 01:00

Close
GMAT Club Daily Prep
Thank you for using the timer - this advanced tool can estimate your performance and suggest more practice questions. We have subscribed you to Daily Prep Questions via email.

Customized
for You

we will pick new questions that match your level based on your Timer History

Track
Your Progress

every week, we’ll send you an estimated GMAT score based on your performance

Practice
Pays

we will pick new questions that match your level based on your Timer History
Not interested in getting valuable practice questions and articles delivered to your email? No problem, unsubscribe here.
Close
Request Expert Reply
Confirm Cancel
SORT BY:
Date
Tags:
Show Tags
Hide Tags
User avatar
Current Student
Joined: 14 Dec 2012
Posts: 580
Own Kudos [?]: 4324 [66]
Given Kudos: 197
Location: India
Concentration: General Management, Operations
GMAT 1: 700 Q50 V34
GPA: 3.6
Send PM
Most Helpful Reply
User avatar
Retired Moderator
Joined: 16 Jun 2012
Posts: 871
Own Kudos [?]: 8554 [19]
Given Kudos: 123
Location: United States
Send PM
User avatar
Manager
Manager
Joined: 11 Aug 2012
Posts: 103
Own Kudos [?]: 328 [5]
Given Kudos: 16
Send PM
General Discussion
User avatar
Manager
Manager
Joined: 07 Nov 2012
Posts: 222
Own Kudos [?]: 912 [1]
Given Kudos: 4
Schools: LBS '14 (A$)
GMAT 1: 770 Q48 V48
Send PM
Re: Company X experienced a significant loss of market share over a number [#permalink]
1
Kudos
Hi -

Firstly. The golden rule of GMAT: It's not about the perfect answer, it's about the best answer. C is clearly the best answer here, so whilst we can debate D, it's not the best answer by anyone's assumption.

The issue with D is that you can't assume: the more percentage, the more increase of market. Thus, determining the maximum percentage of revenue devoted for RnD does not help.
avatar
Manager
Manager
Joined: 12 Sep 2014
Posts: 127
Own Kudos [?]: 139 [1]
Given Kudos: 103
Concentration: Strategy, Leadership
GMAT 1: 740 Q49 V41
GPA: 3.94
Send PM
Re: Company X experienced a significant loss of market share over a number [#permalink]
1
Kudos
I felt that B and D were irrelevant. I was picking between A, C and E. Both A and E are based on how products are doing in the marketplace. However, C seems best because it deals with the marketplace. Remember that with evaluate questions, the answer should be a yes or no and that should be the ultimate decider on the answer.

If competitors left the marketplace, there's no need to spend more money on R&D and the exit of companies allowed an increase in marketshare. However, if non left, then maybe it was the increased R&D expenses that let the market share grow.
User avatar
Manager
Manager
Joined: 19 Sep 2008
Status:Please do not forget to give kudos if you like my post
Posts: 69
Own Kudos [?]: 206 [2]
Given Kudos: 257
Location: United States (CA)
Send PM
Re: Company X experienced a significant loss of market share over a number [#permalink]
1
Kudos
1
Bookmarks
Good question.

Clearly it between C and E, other options are not relevant to the argument.

C. does anyone competes with the companies product for past 2 years?
Yes --> lets keep that R&D budget.
No --> Lets reevaluate. maybe we don't need to spend anymore money.
Best Answer.

E. did our products fared well in marketplace?
Yes --> well R&D budget stays
No --> well R&D budget must stay.
Cant decide anything here. Eliminate this choice.

dangngohai wrote:
Company X experienced a significant loss of market share over a number of years. To strengthen its market position, the company decided two years ago to increase the percentage of total revenue devoted to research and development. The following year, the company's market share increased by 10 percent. In response, company management further increased the amount of money devoted to research and development.

Which of the following investigations is most likely to yield significant information that would help evaluate whether the company's decision to further increase the research and development budget was warranted?

A. Determining how many new products the company has created over the past year
B. Determining the percentage of revenue that other companies devote to research and development
C. Determining whether any of the company's competitors have withdrawn from the market in the past two years
D. Determining the maximum percentage of revenue that the company can devote to research and development without affecting the company's production budget
E. Determining whether all of the company's new products have fared equally well in the marketplace
avatar
Intern
Intern
Joined: 10 Mar 2013
Posts: 11
Own Kudos [?]: 10 [1]
Given Kudos: 78
Send PM
Re: Company X experienced a significant loss of market share over a number [#permalink]
1
Kudos
Correct Answer: C

Because if some competitors have left the market then no need to increase the R&D, and if the market structure remains the same in that case the company has to increase the R&D to increase its market share.

pradeepss:

The problem with Option E is the modifier ALL (Very often GMAT plays on this trick 8-) )

How the YES/NO analysis will help over here:

Did ALL of the NEW products of the company fare well? YES....Great, in that case the increase in the R&D budget is totally warranted

However,
Did ALL of the NEW products of the company fare well? NO.....well, in this case we cannot decide whether the increase in the R&D budget was warranted or not. Because even if one product did well, and increased the market share by little as 10% then it would justify increasing the R&D.

Hope that helps!
Alum
Joined: 19 Mar 2012
Posts: 4341
Own Kudos [?]: 51449 [1]
Given Kudos: 2326
Location: United States (WA)
Concentration: Leadership, General Management
Schools: Ross '20 (M)
GMAT 1: 760 Q50 V42
GMAT 2: 740 Q49 V42 (Online)
GMAT 3: 760 Q50 V42 (Online)
GPA: 3.8
WE:Marketing (Non-Profit and Government)
Send PM
Re: Company X experienced a significant loss of market share over a number [#permalink]
1
Bookmarks
Expert Reply
market share: company's sales/ total industry or market sales

to evaluate whether increase of R&D budget helped increase the companys market share

A.out of scope. new products have no bearing on company market share

B. out of scope. other companies' R&D budget allocation cannot determine if the given company's increased R&D allocation helped improve the market share

D. out of scope. we need to find if the increased R&D
allocation was responsible for the improved market share

E. out of scope.

C-correct, as an "yes' answer to this question identifies an alternate cause for the increase in the company's market share=>improved R&D allocation not responsible. a "no" answer supports company's decision.
Director
Director
Joined: 24 Oct 2016
Posts: 583
Own Kudos [?]: 1321 [0]
Given Kudos: 143
GMAT 1: 670 Q46 V36
GMAT 2: 690 Q47 V38
GMAT 3: 690 Q48 V37
GMAT 4: 710 Q49 V38 (Online)
Send PM
Re: Company X experienced a significant loss of market share over a number [#permalink]
blueseas wrote:
Company X experienced a significant loss of market share over a number of years. To strengthen its market position, the company decided two years ago to increase the percentage of total revenue devoted to research and development. The following year, the company's market share increased by 10 percent. In response, company management further increased the amount of money devoted to research and development.

Which of the following investigations is most likely to yield significant information that would help evaluate whether the company's decision to further increase the research and development budget was warranted?

(A) Determining how many new products the company has created over the past year

(B) Determining the percentage of revenue that other companies devote to research and development

(C) Determining whether any of the company's competitors have withdrawn from the market in the past two years

(D) Determining the maximum percentage of revenue that the company can devote to research and development without affecting the company's production budget

(E) Determining whether all of the company's new products have fared equally well in the marketplace


Official Solution (Credit: Manhattan Prep)



The company's management decided to further increase spending on research and development because the company experienced an increase in market share after the first increase in R & D. The management, however, assumed that the increase in market share resulted directly from the increase in R & D. We are asked which of the listed investigations would allow us to know whether that assumption was valid.

(A) The number of new products does not tell us whether the increase in market share resulted from the increase in R & D.

(B) The percentage of revenue that other companies spend on R & D does not tell us whether the company's increase in market share resulted from its own increase in R & D.

(C) CORRECT. If the company's competitors have dropped out of the market, perhaps the increase in Company X's market share was not the result of the increase in R & D but rather the result of a lack of competition.

(D) The maximum percentage that could be spent on R & D without affecting production does not tell us whether the increase in market share resulted from the increasein R & D.

(E) Knowing whether all of the company's new products have fared equally well will not tell us whether the increase in market share was the result of the increase in R & D.
Manager
Manager
Joined: 16 Jul 2018
Posts: 213
Own Kudos [?]: 68 [0]
Given Kudos: 261
Send PM
Re: Company X experienced a significant loss of market share over a number [#permalink]
Hello guys,
A prompt question regarding this problem, if the question was a little bit different and it said that the company is planning to increase the budget of RD for the next year too, so the question stem would change to ".... to further increase the research and development budget IS warranted?

Then wouldn't answer choice D be correct? (I'm asking because I misinterpret what the question was asking thinking that the next increase will be in a future year not that it has already happened in the previous year)
GMAT Club Bot
Re: Company X experienced a significant loss of market share over a number [#permalink]
Moderators:
GMAT Club Verbal Expert
6920 posts
GMAT Club Verbal Expert
238 posts
CR Forum Moderator
832 posts

Powered by phpBB © phpBB Group | Emoji artwork provided by EmojiOne